Difference between revisions of "2017 AMC 8 Problems/Problem 10"

m (Solution 4)
m (Solution 4)
Line 20: Line 20:
 
==Solution 4==
 
==Solution 4==
 
Let's have three 'boxes'.
 
Let's have three 'boxes'.
One of the boxes must be 4, so <math>\frac{\binom{3}{1} * 3 * 2}{5 * 4 * 3} = \frac{3}{10}</math>
+
One of the boxes must be 4, so <math>\frac{\binom{3}{1} \cdot 3 \cdot 2}{5 \cdot 4 \cdot 3} = \frac{3}{10}</math>
  
 
==Video Solutions==
 
==Video Solutions==

Revision as of 14:06, 11 December 2021

Problem 10

A box contains five cards, numbered 1, 2, 3, 4, and 5. Three cards are selected randomly without replacement from the box. What is the probability that 4 is the largest value selected?

$\textbf{(A) }\frac{1}{10}\qquad\textbf{(B) }\frac{1}{5}\qquad\textbf{(C) }\frac{3}{10}\qquad\textbf{(D) }\frac{2}{5}\qquad\textbf{(E) }\frac{1}{2}$

Solution

There are $\binom{5}{3}$ possible groups of cards that can be selected. If $4$ is the largest card selected, then the other two cards must be either $1$, $2$, or $3$, for a total $\binom{3}{2}$ groups of cards. Then the probability is just ${\frac{{\dbinom{3}{2}}}{{\dbinom{5}{3}}}} = \boxed{{\textbf{(C) }} {\frac{3}{10}}}$

Solution 2 (regular probability)

P (no 5)= $\frac{4}{5}$ * $\frac{3}{4}$ * $\frac{2}{3}$ = $\frac{2}{5}$ this is the fraction of total cases with no fives. p (no 4 and no 5)= $\frac{3}{5}$ * $\frac{2}{4}$ * $\frac{1}{3}$ = $\frac{6}{60}$ = $\frac{1}{10}$ this is the intersection of no fours and no fives. Subtract fraction of no fours and no fives from no fives. $\frac{2}{5} - \frac{1}{10} = \frac{3}{10}$ (C)


Solution 3 (Complementary Probability)

Using complementary counting, $P_\textbf{4 is largest} = 1-P_\textbf{5 is largest} - P_\textbf{3 is largest} = 1- \frac{\dbinom{4}{2}}{\dbinom{5}{3}} - \frac{\dbinom{2}{2}}{\dbinom{5}{3}} = 1- \frac{6}{10} - \frac{1}{10} = \boxed{{\frac{3}{10}}{\textbf{(C)}}}$ -mathfan2020

Solution 4

Let's have three 'boxes'. One of the boxes must be 4, so $\frac{\binom{3}{1} \cdot 3 \cdot 2}{5 \cdot 4 \cdot 3} = \frac{3}{10}$

Video Solutions

See Also:

2017 AMC 8 (ProblemsAnswer KeyResources)
Preceded by
Problem 9
Followed by
Problem 11
1 2 3 4 5 6 7 8 9 10 11 12 13 14 15 16 17 18 19 20 21 22 23 24 25
All AJHSME/AMC 8 Problems and Solutions

The problems on this page are copyrighted by the Mathematical Association of America's American Mathematics Competitions. AMC logo.png